Matematik

Bevis at frekvensfunktionen har maksimum i 𝑥=𝜇

16. juni 2023 af Silvia77 - Niveau: A-niveau

Hej, jeg har den opgave:

Og har fået som tip at "Du kan gennemføre beviset ved at bestemme f’ og derefter løse ligningen f'(x) = 0 med brug af nulreglen."

Og faktisk det er tippen der forvirrer mig utroligt meget. Jeg mener, når man tegne graffen, der er mega tydeligt at funktionen har maksimum i [x=\mu] , men ved ikke hvordan kan jeg bevise det med hjælp af "tippen"

Er der noget kan hjælpe mig? På forhånd tak


Brugbart svar (0)

Svar #1
16. juni 2023 af MandenMedMangeHatte

Du kan ikke vide ud fra grafen om maksimum ligger i \mu eller i \mu + 0.0000001 eller i \mu - 0.0000001. Du er nødt til at finde den afledede og finde ud af hvornår den er lig 0.


Brugbart svar (0)

Svar #2
16. juni 2023 af peter lind

Hvis du differentietere får du -(x-μ)/σ*f(x). Dette kan du gøre som en sammensat funktion hvor -½([(x-μ)/σ]2 er den indre funktion.

Det kan du også se på en anden måde. Da det hele styres af funktionen e-u∧2 er den  størst når u = 0


Svar #3
17. juni 2023 af Silvia77

#1

Du kan ikke vide ud fra grafen om maksimum ligger i \mu eller i \mu + 0.0000001 eller i \mu - 0.0000001. Du er nødt til at finde den afledede og finde ud af hvornår den er lig 0.

du har ret. Jeg mente når man tegne det med f eks geogebra, hvor man kan aflæse værdier rimeligt præcise


Svar #4
17. juni 2023 af Silvia77

#2

Hvis du differentietere får du -(x-μ)/σ*f(x). Dette kan du gøre som en sammensat funktion hvor -½([(x-μ)/σ]2 er den indre funktion.

Det kan du også se på en anden måde. Da det hele styres af funktionen e-u∧2 er den  størst når u = 0

Kan du uddybe lidt den her? Synes funktioner ser lidt mærkeligt ud. Er u=\mu ?


Svar #5
17. juni 2023 af Silvia77

Ok, jeg er kom her: hvis \sigma =1 og \mu = 0 kan udtrykket simpliciceres som f(x)=\frac{e^{-\frac{1}{2}x^2}}{\sqrt{2\pi }}

og så vil f'(x)=-x\frac{e^{-\frac{1}{2}x^2}}{\sqrt{2\pi }}

ved at solve -x\frac{e^{-\frac{1}{2}x^2}}{\sqrt{2\pi }} =0 kommer jeg til at eller -x enten \frac{e^{-\frac{1}{2}x^2}}{\sqrt{2\pi }} er lig med 0, og faktisk, da x er i begge dele er x=0, som er værdi til \mu

Ellers kan jeg bare beregne både f'(x) som f'(x)=0 med CAS, tror jeg (frygter at de bede mig at lave opgaven uden CAS)


Brugbart svar (1)

Svar #6
17. juni 2023 af Anders521

#5 Med tæthedsfunktionen for normalfordelingen er dens afledede givet ved

                                             \begin{align*} f'(x)&= \frac{1}{\sqrt{2\pi}\sigma}\cdot e^{-\frac{1}{2}\cdot (\frac{x-\mu}{\sigma})^2 } \cdot \bigg( \frac{x-\mu}{\sigma^2}\bigg) \\ &= f(x)\cdot \bigg(\frac{x-\mu}{\sigma^2}\bigg)\\ \end{align*}

Da er 

                                             \begin{align*} f'(x)=0 &\Leftrightarrow f(x)\cdot \bigg(\frac{x-\mu}{\sigma^2}\bigg)= 0\\ &\Leftrightarrow \frac{x-\mu}{\sigma^2} =0 \\ &\Leftrightarrow x-\mu=0 \\ & \Leftrightarrow x=\mu \end{align*}


Brugbart svar (1)

Svar #7
17. juni 2023 af ringstedLC

#5: Du får i et tip i opgaven, brug det:

\begin{align*} f'(x)=0 &= \frac{1}{\sqrt{2\,\pi}\,\sigma }\cdot e^{-\frac{1}{2\,\sigma^2}\,\cdot \,(x-\mu)^2}\cdot \left (-\frac{1}{2\,\sigma^2}\right )\cdot (2x-2\mu) \\ &= \frac{1}{\sqrt{2\,\pi}\,\sigma }\cdot e^{-\frac{1}{2\,\sigma^2}\,\cdot \,(x-\mu)^2}\cdot \frac{1}{\sigma^2}\cdot (\mu-x) \\ 0 &= \overset{\neq \;0}{\overbrace{\frac{1}{\sqrt{2\,\pi}\,\sigma ^3}}}\cdot \overset{\neq \;0}{\overbrace{e^{-\frac{1}{2\,\sigma^2}\cdot \,(x-\mu)^2}}}\cdot (\mu-x) \end{align*}


Brugbart svar (1)

Svar #8
17. juni 2023 af oppenede

Din f(x) er sammensat:
\\f(x)=g(h(x)) \\g(x)=(\sqrt{2\pi}\sigma)^{-1}e^x \\h(x)=-0.5((x-\mu)/\sigma)^2
hvor g er monotont voksende, hvorfor f er maksimal ved samme x som h, og h er et andengradspolynomium som er maksimal i toppunktet.


Svar #9
17. juni 2023 af Silvia77

Ok!!! nu jeg tror jeg forstår!!! Tusind tak allesammen :)


Brugbart svar (0)

Svar #10
17. juni 2023 af AMelev

Du kan også se det på den anden måde, som var nævnt i #2
x = 0 ⇔ x2 = 0
x ≠ 0 ⇒ x2 > 0, da et tal i anden ikke kan være negativt

Du ved så, at (\frac{x-\mu}{\sigma})^2\left\{\begin{matrix} =0 & \textup{for}\: x=\mu\\ >0 & \textup{for}\: x\neq \mu \end{matrix}\right.\Leftrightarrow -\frac{1}{2}(\frac{x-\mu}{\sigma})^2\left\{\begin{matrix} =0 & \textup{for}\: x=\mu\\ <0 & \textup{for}\: x\neq \mu \end{matrix}\right.

Altså er (\frac{x-\mu}{\sigma})^2mindst og dermed -\frac{1}{2}(\frac{x-\mu}{\sigma})^2 størst  for x = μ, og da g(t)=\frac{1}{\sqrt{2\pi}\cdot \sigma}\cdot e^ter en voksende funktion er 

f(x)=g(-\frac{1}{2}(\frac{x-\mu}{\sigma})^2)= \frac{1}{\sqrt{2\pi}\cdot \sigma}\cdot e^{-\frac{1}{2}(\frac{x-\mu}{\sigma})^2} også størst for x = μ.


Skriv et svar til: Bevis at frekvensfunktionen har maksimum i 𝑥=𝜇

Du skal være logget ind, for at skrive et svar til dette spørgsmål. Klik her for at logge ind.
Har du ikke en bruger på Studieportalen.dk? Klik her for at oprette en bruger.